Overunity.com Archives is Temporarily on Read Mode Only!



Free Energy will change the World - Free Energy will stop Climate Change - Free Energy will give us hope
and we will not surrender until free energy will be enabled all over the world, to power planes, cars, ships and trains.
Free energy will help the poor to become independent of needing expensive fuels.
So all in all Free energy will bring far more peace to the world than any other invention has already brought to the world.
Those beautiful words were written by Stefan Hartmann/Owner/Admin at overunity.com
Unfortunately now, Stefan Hartmann is very ill and He needs our help
Stefan wanted that I have all these massive data to get it back online
even being as ill as Stefan is, he transferred all databases and folders
that without his help, this Forum Archives would have never been published here
so, please, as the Webmaster and Creator of this Forum, I am asking that you help him
by making a donation on the Paypal Button above
Thanks to ALL for your help!!


A possible violation of the Law of Conservation of Energy

Started by Zetetic, April 14, 2015, 04:59:57 PM

Previous topic - Next topic

0 Members and 1 Guest are viewing this topic.

Zetetic

Hello.

I thought you guys might be interested in this.

It seems to me that you guys have two hurdles to overcome; one theoretical and the other practical.  To successfully build a free energy device you must, one, find an exception to the Law of Conservation of Energy and, two, then get a device mechanically to work with this exception.

I believe I have found a flaw in the Law of Conservation of Energy.  Unfortunately, I don't think this particular flaw I found can lead to a mechanical free energy device.  But, having said that, if I'm right and I found an exception to this law, and thus cleared the theoretical hurdle, I thought you guys still might be interested in this.

I posted my argument at Science Forums.  The original post of my argument was really long and I don't think anyone really read it fully.  The comments that followed were all tangential and didn't address the issue I raised.

I kept thinking about the question and was able to get it down to two pages.  I posted this new formulation of the question in the same thread on April 10th.  There were no responses.  But I know they were reading it based on the "view counter" increasing.  I tried bating them into commenting (e.g. "Silence, generally, means consent").  And finally today, April 14th, I got one response.  But it was meaningless.  The comment was not that there was a logical or factual flaw in my question but rather that had a "poor use of fonts."  And this, to me, seems like another concession that they can't find an actual flaw in my argument.

So, anyway, I thought you guys might be interested in this.  Here is the link to post #21 where I propose the shorted and sharper argument:

http://www.scienceforums.net/topic/87489-a-magnet-is-demagnetized/page-2

If any of you have ever posted anything on Science Forums you know that they love to make snarky comments putting people like you and me in our place when we question the "known Laws of Physics."  Their silence is, I believe, very telling.  It makes me think that I may really be onto something.  I may be wrong (I am all the time), but right now I think I really may have found something.

Again, I don't think it leads the mechanics needed to make an actual free energy device.  But, maybe it does and one of you can find it.  I don't know.

If you do go and read this discussion at Science Forums and if you then choose to comment, please be careful.  If I'm right and I've found a violation of one of their most fundamental beliefs, and that's why they can't respond, I'm sure they would love to lock up the thread and end my blasphemy.  I've been very careful to try to formulate everything in the form of a question and to try to avoid anything that would violate their rules.  All they need is for me or someone else to make a declarative statement like "physics is flawed" or "this proves the Law of Conservation of Energy is wrong" to lock the thread up and move it off to the dustbin of their "Speculations" folder.

So, it's not a free energy device, but I thought it might be related enough to the stuff you guys are into that you might be interested.

And that's it.  (Again, I may be wrong but right now I think I really may have found something.)

Take care.



ayeaye

I'm likely not into all the scientific subtleties of this, and what laws of physics it may or may not violate. Going into equations would take aeons there i guess, so i prefer simple principles. Thus sorry if my question is somewhat naive. So what are you going to do, achieve a continuous movement by demagnetizing a permanent magnet using heat? This may well be possible, but then does not heating the magnet take energy?

There may be overunity in permanent magnets, but by my experiments i found that this is not enough for continuous rotation. Overunity is not the same as permanent motion. But can you say where the energy on the image below comes from? I don't say that the law of conservation of energy is violated, i just don't know where the energy comes from. And for making  free energy devices, i also don't see that it is important to answer to that question. I did the experiment of that, see this video https://archive.org/details/Flcm3 . This experiment was done just so that one can repeat it, and see oneself whether the results are the same. Just make sure to use magnets of the same size and strength, and a disc with a good ball bearings.

The result was that when starting to move from almost the same point, the disc moved some two times more by the chain of magnets, than in the opposite direction. This should not be so when all the energy came from moving the disc to the starting position, and all the movement was only caused by repulsion of the first magnet. This does not provide a continuous rotation with permanent magnets or electromagnets, so for that it is useless. Which does not mean that such effect may not cause overunity when charged particles move in a solid state device.

Thank you for your attention.

Zetetic


hey ayeaye ...

Thanks for the video link.  Cool design!

I watched the video you linked to (Field lines chain motor 3/4), as well as the links to the previous video (Field lines chain motor 1/4) and the next video (Field lines chain motor 4/4).

(Is there also a "Field lines chain motor 2/4" and I just missed it?)

The "Field lines chain motor 1/4" worked as I would have guess it would.  Whether you have one magnet that is capable of moving or several aligned magnets capable of moving, I would expect them to move so that the pole of the moveable magnet (or magnets) moves away from where it is repulsed by the external and fixed magnet and to where it is attracted to the external and fixed magnet (the one in your hand).

I was surprised by the results of "Field lines chain motor 3/4."  I did not expect the movable magnets to spin past, and to spin so far past, the end of the chain where the pole of the moveable magnets is attracted to the external and fixed in place magnet.  That was interesting.

What my guess is that you have found a magnetic arrangement, in "Field lines chain motor 3/4", where the repulsed starting point is a greater force than the attractive point at the end of the row (curved row) of magnets.  And so, even though the row of magnets are attracted to the external fixed magnet (as was seen in "Field lines chain motor 1/4") in this arrangement the force and the resultant velocity from the repulsed starting point is enough to break free and move beyond the greatest point of attraction between the moveable magnets and the external fixed magnet in "Field lines chain motor 3/4".  So, I was surprised by the results, but it makes sense that such a configuration as you found could be made.

"Field lines chain motor 4/4" did work as I would have expected.  By going from a row of magnets with a south end and a north end ("Field lines chain motor 3/4") to a full circle ("Field lines chain motor 4/4") means going from a system where there is a point of greater repulsion and a point of greater attraction to a system where there is no point of greater repulsion and no point of greater attraction (assuming a perfect physical system with no imperfections).  And what I would expect in this case is, after the moveable magnets have been set in motion (by your hand), for them to not continue in motion by a force from the arrangement (since there is no greater point of repulsion for them to get away from and no greater point of attraction for them to move towards), but rather move for a while due to having been set in motion by your hand and then eventually come to a stop due to friction (your device, like all real world devices, is not frictionless and friction brings motion to a stop (while generating thermal energy).)  And the fact that your device in "Field lines chain motor 4/4" tended to come to a stop at the same point in the circle of magnets also is what I would have expected.  In a hypothetical perfect circle of magnets I would expect it to come to a stop at different random points.  However, in a real world device (where there will always been some slight differences, imperfections, in the manufactured magnets and some slight differences (even if only very very tiny) in the size and shape of the gaps between the magnets) it is expected that there would be some point around the circle of magnets where the attraction of the circle of magnets is greater (even if only slightly greater) with the external fixed magnet.

I know the results of "Field lines chain motor 3/4," where the curved row of magnets moved well beyond the external fixed magnet, might seem like this would lead to continuous motion forever when then moving from this configuration to a complete circle of magnets, but I think that by going from a row to a complete circle you are eliminating the dynamic (a point of greater repulsion and a point of greater attraction) that caused the movement in "Field lines chain motor 3/4."

I think that if you were able to physically build a truly perfect "Field lines chain motor 4/4" (with no slight imperfections or variations) that it would come to a stop, due to friction, at random points, while if you were to build such a design (with real world materials and real world placements that will always be something less, even if only slightly less, that perfect) then you'd see it come to a stop in one or two (or so) places more often than not.

But ... this is just my opinion.

---

If you're interested in the Law of Conservation of Energy and how it relates to free energy devices, I can explain it to you in a couple of very short and common sense language paragraphs (no Physics jargon needed, no math needed, and I promise it will only be a small fraction of the length of my review of you design).

So, if you're up for that, brief, discussion/explanation ... let me know.

---

Again, cool design!  Thank you for showing it to me.  I hope my analysis of it is of some use.

Take care!

- Zet 



(PS:  While I believe I understood what you were going for in the videos, I'm sorry, but I don't think I quite understood what you were trying to explain with your drawing.)




ayeaye

What i was trying to explain with my drawing is this. A north pole of another magnet is moving through the field of the magnet, which is as it is drawn there. And it moves in the direction shown by the arrow. Now when this pole of the magnet is where the field lines are horizontal, then it moves by the field lines, this is what field lines mean. And when moving by the field lines, it gets a propulsion, to the direction where these field lines go.

The only thing which may work against it, is a repulsion, that is going through the field lines when their direction is opposite to the movement. But as can be seen on the drawing, this never happens, because in the worst case the field lines are perpendicular to the movement, and never opposite to the movement. Which means that the moving magnet gets extra speed when moving through the magnetic field of the other magnet, and thus an additional energy. This should not happen when there is no overunity, the pole of the magnet should go through the field of another magnet without getting any additional energy. Thus the additional energy should come from somewhere, the question is from where.

The reason why this happens is that magnetic field is not symmetric. It were symmetric if it were just radial lines coming from the magnet, like a field around a sphere. But it is not like that, and thus magnetic field is not symmetric. And every asymmetric field can in theory do work.

Zetetic




ayeaye,


I guess I'm not sure how you have the two magnets oriented towards one another in the drawing.

In your videos I thought your magnets (the set moving magnets (row and full circle) and the fixed magnet) were orientated relative one another as they are in A in the drawing below.  (But, of course, in your videos it's the other magnet or magnets that is in motion and the other magnet that is fixed.)

And so, this is how I thought they were also oriented in your drawing.

But, now, based on your last response, I think maybe in your drawing you have the orientation different, as shown in B in the drawing below.

I hope to understand what you're getting at, but we need to clear up my understanding of your point.

A?
B?
Or, something else?


- Zet